Está en la página 1de 8

D OMAINE : Arithmtique

N IVEAU : Avanc
C ONTENU : Cours et exercices

A UTEUR : Igor K ORTCHEMSKI


S TAGE : Grsillon 2011

Ordre modulo n, LTE

Nous rappelons ici quelques lments vus dans le cours darithmtique


avanc. On suppose connus les rsultats classiques darithmtique (congruences, inversibilit modulo n, petit thorme de Fermat, fonction dEuler ;
pour ces notions voir le cours darithmtique sur le site dAnimath). Les solutions aux exercices se trouvent la fin du cours.
- Ordre dans Z/nZLe thorme suivant nous permettra de dfinir la notion dordre.
Thorme 1. Soient a, n des entiers naturels. Les propositions suivantes sont
quivalentes :
1. a et n sont premiers entre eux.
2. Il existe un entier k tel que :
ak 1

modulo n.

Dmonstration. Prouvons dabord le sens direct. Daprs le thorme relatif


la fonction dEuler, il suffit de prendre k = (n).
Donnons une autre preuve plus simple qui nutilise pas ce thorme. Comme
lensemble des rsidus modulo n est fini, il existe deux entiers distincts r < s
tels que :
as ar modulo n.
Alors n divise ar (asr 1), donc aussi asr 1 car a et n sont premiers entre
eux. Il suffit donc de prendre k = r s.
Pour le sens rciproque, raisonnons par labsurde en supposant que d =
pgcd(a, n) > 1. Par hypothse, il existe un entier r tel que ak + rn = 1. Cela
implique que d divise 1, ce qui est contradictoire.
1

Dfinition 2. Soient a, n des entiers naturels premiers entre eux. On appelle


ordre de a modulo n le plus petit entier non nul not n (a), vrifiant :
an (a) 1

modulo n.

Lorsquil ny a pas dambiguit possible, on notera (a) la place de n (a).


Daprs le thorme 1, cette dfinition a bien un sens. Le thorme suivant illustre un des intrts de cette dfinition.
Thorme 3. Soient a, n des entiers naturels premiers entre eux et k un entier
vrifiant :
ak 1 modulo n.
Alors (a) divise k.
Dmonstration. Par labsurde, supposons que (a) ne divise pas k. Soit (a) =
qk + r la division euclidiennt de (a) par k, avec 0 < r < k 1. Alors :
1 a(a) modulo n
q
(ak ) ar modulo n
ar modulo n.
Ceci contredit le caractre minimal de (a) et permet de conclure.
Corollaire 4. Soient a, n des entiers naturels premiers entre eux. Alors lordre
de a divise (n). En particulier, lorsque n = p et premier, lordre de a divise
p 1.
Dmonstration. Ceci provient du thorme prcdent et du fait que a(n)
1 modulo n.
Ce corollaire est en particulier utile lorsquon veut cherche lordre dun entier modulo n la main : il suffit de test les diviseurs de (n).
Remarque 5. Attention ; il faut bien garder de croire que si p et premier et
0 < a 6 p 1, alors lordre de a est p 1. Comme nous venons de le voir, sil
est vrai que lordre de a divise p 1, il ny a pas en gnral galit : il suffit
par exemple de prendre p = 7 et a = 2 pour sen convaincre. Cependant, le
thorme suivant donne un rsultat allant en ce sens.
Exercice 1 Existe-t-il des entiers n > 1 tels que 9 divise 7n + n3 ?
Exercice 2
2

(i) Trouver tous les entiers n > 1 tels que n divise 2n 1.


(ii) Trouver tous les entiers n > 1 impairs tels que n divise 3n + 1.
Exercice 3 Trouver tous les entiers m, n > 1 tels que mn divise 3m + 1 et mn
divise 3n + 1.
Finissons par quelques rsultats algbriques concernant lordre. Pour des
entiers a, n on note a n le PGCD de a et de n.
Proposition 6. Soient a, b, n des entiers avec an = 1 et bn = 1. Supposons
que (a) (b) = 1. Alors (ab) = (a)(b).
Dmonstration. Il est clair que (ab)(a)(b) = 1. On en dduit que (ab) divise
(a)(b).
Soit maintenant k > 1 tel que (ab)k = 1 et montrons que k > (a)(b).
En levant la puissance (a), il vient bk(a) = 1. On en tire que (b) divise
k(a), et daprs lhypothse cela implique que (b) divise k. On montre
de mme que (a) divise k. Daprs lhypothse, on en dduit encore que
(a)(b) divise k, do le rsultat.
Si a et n sont premiers entre eux, nous conseillons au lecteur dessayer de
dterminer lordre de ak modulo n en fonction de lordre de a modulo n.
- Inversibilit dans Z/nZ
Nous venons de voir que le fait que ak soit congru 1 modulo n donnait
une relation de divisibilit entre k et lordre de a modulo n. Nous allons maintenant voir que des informations similaires peuvent tre dduites dune relation de type ak bk mod n. cet effet, rappelons le rsultat suivant :
Thorme 7. Si a n = 1, il existe un entier b tel que ab 1 mod n. Cet entier
est appel inverse de b modulo n et sera not b1 ou 1/b.
Dmonstration. Donnons deux preuves de ce rsultats.
Premire dmonstration : il suffit de prendre n = an (a)1 .
Seconde dmonstration : daprs le thorme de Bzout, il existe des entiers
b, d tels que ab+nd = 1. En rduisant modulo n, on obtient ab 1 mod n.
Exercice 4 Soient p, q deux nombres premiers tels que q divise 3p 2p . Montrer
que p divise q 1.
Exercice 5 Trouver les a, n > 1 tels que ((a + 1)n an )/n est un entier.
3

Exercice 6 Soient a, b > 1 impairs tels que a + b = 2 avec > 1. Montrer


quil ny a pas dentiers k > 1 tels que k2 divise ak + bk .
- Racines primitives Considrons deux entiers a et n premiers entre eux. Nous avons vu n (a),
lordre de a modulo n, divise (n). Il est donc naturel de se demander sil
existe des entiers a tels que n (a) = (n).
Dfinition 8. Soient a, n deux entiers premiers entre eux. On dit que a est une
racine primitive modulo n si a (n) = (n).
Si a est racine primitive modulo n, on voit que les restes modulo n des
entiers de lensemble {1, a, a2 , . . . , a(n)1 } sont tous distincts.
Remarquons tout de suite quil nexiste pas forcment de racines primitives : il est facile de voir quil ny a pas de racine primitive modulo 6. titre
culturel, mentionnons le rsultat suivant :
Thorme 9. Il existe une racine primitive modulo n si, et seulement si, n =
2, 4, pk ou 2pk avec p un nombre premier impair et k > 1.
Nous verrons en TD que si n est divisible par au moins deux nombres premiers impairs distincts, alors il nexiste pas de racine primitive modulo n.
Nous allons maintenant dmontrer un cas particulier du thorme prcdent.
Sa dmonstration peut tre saute en premire lecture, mais le rsultat est
retenir.
Proposition 10. Si p est premier, il existe une racine primitive modulo p.
Dmonstration. Montrons dabord que si q est un nombre premier tel que q
divise p 1 avec > 1, alors il existe un lment dordre q modulo p. cet
effet, pour x = 1, . . . , p 1 on introduit :
p1

yx = x q .

En particulier, daprs le petit thorme de Fermat, yqx 1 mod p. On en


dduit que lordre de yx modulo p divise q . crivons donc :
p (yx ) = qnx .
Notons ensuite nmax = max{nx ; x = 1, . . . , p 1}. Il suffit de montrer que
nmax = (ceci impliquera quil existe un lment dordre q modulo p). Pour
cela, on introduit le polynme :
p1

P(X) = X q q
4

nmax

et remarque que pour x = 0, 1, . . . , p 1 :


P(x) = yqx

nmax

1 mod p

car p (yx ) divise qnmax .


Il en dcoule que le polynme P(X) a p1 racines distinctes dans Z/pZ, qui
est un corps (si vous ne savez pas ce que cest quun corps, vous pouvez sauter
cet argument). On en dduit que le degr de P vaut au moins p 1, de sorte
que nmax > . Comme il est clair que nmax 6 , on en dduit que nmax = .
Revenons maintenant la preuve du thorme. Soit p 1 = q1 1 qk k la
dcomposition en facteurs premiers de p 1. Daprs ce qui prcde, pour
1 6 i 6 k il existe un lment xi dordre qi k . En utilisant la proposition 6, il
vient :
(x1 x2 xk ) = (x1 )(x2 ) (xk ) = q1 1 qk k = p 1.
Llment x1 x2 xk est donc dordre exactement p 1.
Remarquons que la preuve prcdente ne sapplique pas au cas gnral
Z/nZ, car ce dernier anneau nest un corps que pour n premier.
Remarque 11. La preuve prcdente montre que si p est premier et que si d
divise p1, alors il existe un lment dordre d. Il est possible de montrer quil
y a alors exactement (d) lments dordre d.
Exercice 7 Trouver tous les entiers n tels que 19 divise 23n+4 + 32n+1 .
Exercice 8 Soient a, b, n des nombres entiers strictement positifs avec a > b.
Montrer que n divise (an bn ).
- LTE : Lifting The Exponent Nous concluons ce cours par un thorme extrmement utile en arithmtique. Pour un entier n et un nombre premier p, on note vp (n) lexposant de la
plus grande puissance de p divisant n.
Thorme 12. Soit p un nombre premier impair. Soient a, b des nombres entiers (non ncessairement positifs) et un entier n > 1. On suppose que p divise
a b mais que p ne divise ni a ni b. Alors :
vp (an bn ) = vp (a b) + vp (n).
5

Ce thorme doit tre connu. Nous renvoyons au texte suivant :


http ://www.artofproblemsolving.com/Resources/Papers/LTE.pdf
pour une preuve, des extensions au cas p = 2 et de nombreux exemples
dapplication. Nous ne pouvons quencourager fortement le lecteur lire attentivement ce dernier texte.
Exercice 9 Soient a, n deux entiers strictement positifs et p un nombre premier
impair tel que ap 1 mod pn . Montrer que a 1 mod pn1 .
Exercice 10 Soit k un entier strictement positif. Trouver tous les entiers strictement positifs n tels que 3k divise 2n 1.
- Solutions des exercices du cours Solution de lexercice 1 Soit n > 1 tel que 9 divise 7n + n3 . Comme un cube est
congru 0, 1 ou 1 modulo 9, on en dduit que n6 1 mod 9 et donc que
72n 1 mod 9. Or lordre de 7 modulo 9 est 3. On en dduit que 3 divise 2n.
Ainsi 3 divise n. Il faudrait donc que 3 divise 7n , ce qui est absurde. Il ny a
donc pas de tels entiers.
Solution de lexercice 2
(i) Soit n > 1 tel que n divise 2n 1. Il est clair que n est impair. Soit p le
plus petit facteur premier de n, qui est donc impair. Alors 2n 1 mod p.
Soit lordre de 2 modulo p. Alors divise n. Dautre part, daprs le
petit thorme de Fermat, 2p1 1 mod p. Ainsi divise p 1. Daprs
la condition sur p, on a ncessairement = 1. Alors 2 1 mod p, ce qui
est absurde. On a donc forcment n = 1.
(ii) Soit n > 1 tel que n divise 3n + 1. Soit p le plus petit facteur premier
de n, qui est donc impair, qui vrifie donc p > 3. Alors 32n 1 mod p.
Soit lordre de 3 modulo p. Alors divise 2n. Dautre part, daprs
le petit thorme de Fermat, 3p1 1 mod p. Ainsi divise p 1. On
en dduit que divise pgcd(2n, p 1). Daprs la condition sur p, on a
ncessairement = 1 ou 2. Dans le premier cas de figure, 3 1 mod p et
donc p = 2, ce qui est exclu. Dans le deuxime cas, 32 1 mod p et donc
p divise 8, ce qui est exclu galement. On en dduit que n = 1.
Solution de lexercice 3 On suppose m, n > 2. Soit p le plus petit diviseur de n.
Alors 32n 1 mod p. Soit lordre de 3 modulo p. Alors divise 2n. Dautre
part, daprs le petit thorme de Fermat, 3p1 1 mod p. Ainsi divise p1.
6

On en dduit que divise pgcd(p 1, 2n). Daprs la condition sur p, on a


ncessairement = 1 ou 2. Dans le premier cas de figure, Dans le premier cas
de figure, 3 1 mod p et donc p = 2. Dans le deuxime cas, 32 1 mod p et
donc p = 2. On en dduit que n est pair. On montre de mme que m est pair.
Alors 4 divise 3mn + 1, ce qui nest pas possible car mn est pair.
Il reste examiner le cas o m ou n vaut 1 et il vient que les solutions sont
(1, 1), (1, 2) et (2, 1).
Solution de lexercice 4 Il est clair que q > 5. Notons lordre 3/2 modulo q
(rappelons que 1/2 dsigne linverse de 2 modulo q). Alors divise p, donc
= 1 ou p. Le premier cas ntant pas possible, on a donc = p. Or daprs
le petit thorme de Fermat, (3/2)q1 1 mod q. On en tire que divise q 1,
do le rsultat.
Solution de lexercice 5 Supposons que n > 2. Soit p le plus petit facteur premier de n. Alors p divise (a + 1)n an . En dautres termes, ((a + 1)/a)n 1
mod p. Soit lordre de (a + 1)/a modulo p. Alors divise n. Dautre part,
daprs le petit thorme de Fermat, ((a + 1)/a)p1 1 mod p de sorte que
divise p 1. Daprs la condition sur p, ncessairement = 1. Ceci implique
a + 1 a mod p, ce qui est absurde.
Les solutions sont donc n = 1 et a quelconque.
Solution de lexercice 6 Raisonnons par labsurde et considrons un entier k > 1
tel que k2 divise ak + bk . En raisonnant modulo 4 on voit que k est impair.
Comme a + b est une puissance de 2, il en dcoule que a et b sont premiers
entre eux. Soit p le plus petit facteur premier de k qui est donc diffrent de 2
et ne divise ni a, ni b.
Soit lordre de a/b modulo p. Comme dans les exercices prcdents, on
voit que divise 2k ainsi que p 1. Daprs la condition sur p, on a ncessairement = 1 ou 2. Dans le premier cas de figure, a b mod p et donc
2ak 0 mod p, ce qui est absurde. Dans le deuxime cas de figure, a2 b2
mod p. Ainsi p divise (a b)(a + b). On a vu que a b mod p ntait pas
possible. Mais comme a + b = 2 , on ne peut pas non plus avoir a + b 0
mod p car p est impair. Ceci conclut la solution.
Solution de lexercice 7 Les conditions de lnonc impliquent que 9n 8n mod
19. Mais linverse de 8 modulo 19 est 12. On en dduit que 13n 108n (9
8)n 1 mod 19. Or 13 est racine primitive modulo 19. Les entiers recherchs
sont donc lensemble des multiples de 18.
Solution de lexercice 8 Traitons dabord le cas o a et b sont premiers entre eux.
7

Alors a et b sont premiers avec an bn et il est clair que lordre de a/b modulo
an bn est n. On en dduit que n divise (an bn ).
Si d > 1 est le PGCD de a et de b, notons u = a/d et v = b/d de sorte que
u et v sont premiers entre eux. Daprs ce qui prcde, n divise (un vn ). En
utilisant la formule exprimant (n) en fonction des facteurs premiers de n, on
voit que (un vn ) divise (dn (un vn )) = (an bn ), ce qui conclut.
Solution de lexercice 9 Il est clair que a et p sont premiers entre eux. Daprs
le petit thorme de Fermat, ap1 1 mod p. Comme ap 1 mod p, on en
dduit que a 1 mod p. On peut donc utiliser LTE et on obtient :
vp (a 1) + 1 = vp (a 1) + vp (p) = vp (ap 1).
Par hypothse, le dernier terme est suprieur ou gal n. Il en dcoule que
vp (a 1) > n 1, ce quil fallait dmontrer.
Solution de lexercice 10 Soit k tel que 3k divise 2n 1. En raisonnant modulo
3, on voit que n est pair. crivons donc n = 2m avec m > 0. Alors 3k divise
4n 1. Comme 3 divise 4 1, on peut appliquer LTE :
v3 (4 1) + v3 (n) = v3 (4n 1) > k.
On en dduit que v3 (n) > k 1. Ainsi 2 3k1 divise n.
Rciproquement, le mme raisonnement nous donne que 3k divise 2n 1 si
2 3k1 divise n.

También podría gustarte